Đến nội dung

Le Dinh Hai nội dung

Có 64 mục bởi Le Dinh Hai (Tìm giới hạn từ 12-05-2020)



Sắp theo                Sắp xếp  

#551383 $\frac{(x+y+z)^4}{x^2+y^2+z^2}\geq 27$

Đã gửi bởi Le Dinh Hai on 04-04-2015 - 17:13 trong Bất đẳng thức và cực trị

Cho x,y,z>0 thỏa $x+y+z=xy+yz+zx$. Chứng minh $\frac{(x+y+z)^4}{x^2+y^2+z^2}\geq 27$

x+y+z=xy+xy+zx

Nên (x+y+z)^2=x^2+y^2+z^2+2xy+2yz+2zx=x^2+y^2+z^2+2x+2y+2z

=>x^2+y^2+z^2=(x+y+z)^2-2x-2y-2z

Đặt x+y+z=a,ta có:

x^2+y^2+z^2=a^2--2a;(x+y+z)^3=a^3;

Thay vào BDT ban đầu,ta có:

a^3-27a+54 >=0(a<>0;a<>2)

=>(a-3)(a^2+3a-18)>=0

Xét a>=3,ta có

a-3 >=0;a^2+3a-18>=0;(1)

Xét a<3;a<>2.ta có:

a-3<0;a^2+3a-18<0;(2)

Từ (1),(2) => dpcm

Dấu = xảy ra tại a=3<=>x+y+z=3




#551551 Tìm số vở và số nhóm.

Đã gửi bởi Le Dinh Hai on 05-04-2015 - 11:08 trong Đại số

Nếu đến nhóm n mà lấy 1 phần 9 thì còn 8 phần 9 nữa.thế số vở sao chia hết được.




#551564 Chứng minh rằng nếu $2^{n}=10a+b$ thì tích ab chia hết ch...

Đã gửi bởi Le Dinh Hai on 05-04-2015 - 12:14 trong Số học

Cho số tự nhiên $n> 3$. Chứng minh rằng nếu $2^{n}=10a+b$ (a, b$\in \mathbb{N}$, 0<b<10) thì tích ab chia hết cho 6.

Vì $2^{n}$ có tận cùng là 2;4;6;8 nên b có thể là 2;4;6;8.

Với $b=2$ => $2^{n}$ có dạng $2^{4x+1}$

Nên $2^{4x+1}=10a+2$

=>$2^{4x+1}-2=10a$

=>$2(2^{4x}-1)=10a$

=>$5a=2^{4x}-1$

=>$5a=16^{x}-1$

=>$5a=15A$

=>a chia hết cho 3

=>ab chia hết cho 6.

Tương tự,với $b=4;6;8$=>đpcm




#551571 Biết x, y là nghiệm của phương trình $x^{2}-2xy+2y^{2...

Đã gửi bởi Le Dinh Hai on 05-04-2015 - 12:34 trong Đại số

Thế chắc đề bị sai rồi.Vì với 2 cách giải khác nhau nhưng có chung kết quả thì khó lòng sai lắm!




#551583 $T=\sqrt{\left({x-a}\right)\left({b-x}\right)}+...

Đã gửi bởi Le Dinh Hai on 05-04-2015 - 14:54 trong Bất đẳng thức và cực trị

Áp dụng BĐT Cauchy cho 2 số không âm ta được:

 

$T=\sqrt{(x-a)(b-x)}+\sqrt{(x-c)(d-x)} \leq \frac{x-a+b-x}{2}+\frac{x-c+d-x}{2}=\frac{b-a+d-c}{2} \leq 0$

 

Dấu = xảy ra $\Leftrightarrow a=b=c=d\geq 0$

Theo mình thì $T\geq 0$ chứ.Dấu = xảy ra tại $a=b=c=d=x$




#551591 Có 3 chiếc bát trên bàn , mỗi chiếc đựng 1 loại quả khác nhau.Hỏi bát đựng ca...

Đã gửi bởi Le Dinh Hai on 05-04-2015 - 15:10 trong Toán rời rạc

Bài 2: Sau lần 1 còn  $\frac{1}{2}$ bình.Sau lần 2 còn $(\frac{1}{2})(1-\frac{1}{3})=\frac{1}{3}$ bình....

...sau 9 lần sẽ còn $\frac{1}{10}$ bình.




#551599 Có 3 chiếc bát trên bàn , mỗi chiếc đựng 1 loại quả khác nhau.Hỏi bát đựng ca...

Đã gửi bởi Le Dinh Hai on 05-04-2015 - 15:34 trong Toán rời rạc

3/ Đặt Tg là x; Hv là y đối với Hs1;z,t với  Hs2;ta có;

$89+16+69+60+8+88+10x+y=88+80+9+69+91+68+10t+z$

$=>xy+330=tz+405$

$=>xy=tz+75$

Ta thấy,$x=1 => z=1$

$x=6 => z=9$

$x=8 =>z=8$

$x=9 => z=6$

Tương tự với y và t.

Chỉ có $91-16=75$

nên $x=9$;$y=1$

=> Kết quả:421




#551606 Tìm n nguyên dương thỏa mãn

Đã gửi bởi Le Dinh Hai on 05-04-2015 - 15:54 trong Các dạng toán khác

$\sqrt{\left ( 3+2\sqrt{2} \right )^{n}}+\sqrt{\left ( 3-2\sqrt{2} \right )^{n}}= 6$

$\sqrt{\left ( 3+2\sqrt{2} \right )^{n}}+\sqrt{\left ( 3-2\sqrt{2} \right )^{n}}= 6$

<=>$\sqrt{left(1+\sqrt{2}\right)^{2n}}+\sqrt{left(1-\sqrt{2}\right)^{2n}}=6$

<=>$(1+\sqrt{2}\right)^{n}+(-1+\sqrt{2}\right)^{n}=6$

......

......

=>$n=2$




#575137 Cho tam giác $ABC$...Chứng minh rằng $AI$ vuông góc...

Đã gửi bởi Le Dinh Hai on 24-07-2015 - 21:49 trong Hình học

Cho tam giác $ABC$. Trên các cạnh $AB,AC$, dựng ra phía ngoài các tam giác $ABE$,$ACF$ vuông cân tại $A$. Gọi $I$ là trung điểm cạnh $BC$. Chứng minh rằng $AI$ vuông góc $EF$

Hạ $BH$,$CK$ vuông góc $AI$.

=>$BH$=$CK$

Kéo dài $IA$ cắt $EF$ tại D

Ta có$\angle BAI + \angle DAE = 90^{\circ}$

Lại có$\angle BAI + \angle ABH = 90^{\circ}$

=> $\angle ABH = \angle DAE

Tương tự,ta có $\angle CAI =\angle DAF$

Vẽ tam giác $CKM$ = tam giác $BHA$

=>Tam giác $EAF$ = tam giác $MCA$

=>$\angle ABH = \angle FEA$

=>$\angle FEA + \angle EAD =  90^{\circ}$

=>(đpcm)




#575146 $4a+\frac{1}{a}$

Đã gửi bởi Le Dinh Hai on 24-07-2015 - 22:09 trong Bất đẳng thức và cực trị

Một đề mình đã đọc người ta bảo tìm $MAX$ và $a,b,c$ dương

Ta có bất đẳng thức quen thuộc . 

$\frac{1}{x}+\frac{1}{y} \geq \frac{4}{x+y}$

$\frac{ab}{c+1}+\frac{bc}{a+1}+\frac{ca}{b+1}=\frac{ab}{a+c+b+c}+\frac{bc}{b+a+c+a}+\frac{ca}{a+b+c+b}\leq\frac{1}{4}(\frac{a}{a+c}+\frac{b}{b+c}+\frac{b}{a+b}+\frac{c}{c+a}+\frac{c}{c+b}+\frac{a}{a+b})=\frac{3}{4}$

Sai rồi bạn

$\frac{4ab}{(a+c)(b+c)} \leq \frac{a}{a+c} + \frac{b}{b+c}$

Với lại dấu $"="$ xảy ra tại $a=b=c$ =>$Sum = \frac{1}{4}$




#575229 $4a+\frac{1}{a}$

Đã gửi bởi Le Dinh Hai on 25-07-2015 - 13:48 trong Bất đẳng thức và cực trị

Với đề bài $Max$

Ta có$\sum \frac{ab}{c+1} = \sum \frac{ab}{a+c+b+c} =\sum \frac{1}{\frac{a+c}{ab}+\frac{b+c}{ab}}\leq \sum \frac{1}{4}\left ( \frac{ab}{a+c}+\frac{ab}{b+c} \right )=\frac{1}{4}\left ( a+b+c \right )=\frac{1}{4}$

Dấu $"="$ xảy ra tại $a=b=c=\frac{1}{3}$




#575372 Cho $a,b,c>0$ , $a+b+c=1$. Cm $(a+\frac...

Đã gửi bởi Le Dinh Hai on 25-07-2015 - 20:54 trong Đại số

mình chưa hiểu đoạn $a+b+c\geq 3\sqrt[3]{abc}\Leftrightarrow abc\leq \frac{1}{27}$ ( lớp 7 chưa học bất đẳng thức cu-si), giải thích giùm mình nha tại não hơi bé

Ta có:$\left (\sqrt{a}-$a+b\geq 2\sqrt{ab}$\sqrt{b}\right )^{2}\geq 0$

=>$a+b\geq 2\sqrt{ab}$   (1)

Tương tự:$c+\sqrt[3]{abc}\geq 2\sqrt{c\sqrt[3]{abc}}$   (2)

Từ (1),(2) =>$a+b+c+\sqrt[3]{abc}\geq 2\sqrt{ab}+2\sqrt{c\sqrt[3]{abc}}\geq 4\sqrt[4]{abc\sqrt[3]{abc}}=4\sqrt[3]{abc}$

=>$a+b+c\geq 3\sqrt[3]{abc}$




#575406 Tỉ số lượng giác của góc nhọn

Đã gửi bởi Le Dinh Hai on 25-07-2015 - 21:55 trong Hình học

Cho tam giác nhọn ABC có: BC=a;  CA=b;  AB=c. R là bán kính đường tròn ngoại tiếp

Cmr: $\frac{a}{sinA}=\frac{b}{sinB}=\frac{c}{sinC}=2R$

Vẽ tam giác $BCD$ vuông tại C nội tiếp đường tròn trên.

Ta có $BD.sinBDC=BC$

<=>$2R=\frac{a}{sinBDC}$

mà $\angle BDC = \angle BAC$

=>$\frac{a}{sinA}=2R$

Tương tự ta có $đpcm$




#575421 CMR: Có ít nhất 3 số trong chúng là độ dài 3 cạnh của 1 tam giác

Đã gửi bởi Le Dinh Hai on 25-07-2015 - 22:25 trong Số học

Cho 10 số thực $x_{1};x_{2};...;x_{10}$ thuộc đoạn $[1;55)$

CMR: Có ít nhất 3 số trong chúng là độ dài 3 cạnh của 1 tam giác

Giả sử không tồn tại 3 số nào thỏa mãn

Gọi 3 số bất kì trong dãy là $a,b,c(a\leq b\leq c)$

Ta có $a+b\leq c$

Theo thứ tự $x_{1}\leq x_{2}\leq ...\leq x_{10}$ thì để $x_{10}$ nhỏ nhất ta có

$x_{1}=1;x_{2}=2;x_{3}=3;x_{4}=5,...x_{10}=55$ (vô lí)

=>$(đpcm)$




#575541 CMR: Có ít nhất 3 số trong chúng là độ dài 3 cạnh của 1 tam giác

Đã gửi bởi Le Dinh Hai on 26-07-2015 - 14:12 trong Số học

bạn giải thích rõ hơn được không 

Để không tồn tại tam giác thì $a+b\leq c$

$x_{1}+x_{2}\leq x_{3}$

Tương tự thì sum của 2 số bé luôn $\leq$ số lớn




#575544 Tìm $n$ tự nhiên sao cho số $2^{n}-1\vdots 7$

Đã gửi bởi Le Dinh Hai on 26-07-2015 - 14:31 trong Số học

Tìm $n$ tự nhiên sao cho số $2^{n}-1\vdots 7$

Với $n=3k$ $\left ( k\epsilon \mathbb{N} \right )$

Ta có $2^{n}-1= 2^{3k}-1^{k}=\left ( 2^{3} -1\right )M=7M \vdots 7$

suy ra $2^{3k}\equiv 1(mod7)$

$2^{3k+1} \equiv 2(mod7)$

và $2^{3k+2}\equiv 4 (mod7)$

Suy ra $n=3k$




#575552 Ảo ảnh thị giác lừa bộ não

Đã gửi bởi Le Dinh Hai on 26-07-2015 - 14:54 trong Toán học lý thú

có ví dụ thực tế của hình 

Khi ngủ nhé! Bạn nhìn qua màn tập trung nhìn vào những vật bên ngoài thì thấy màn gần như biến mất




#575674 $A=\frac{5x^{2}-7x+1}{\left | 3x-2...

Đã gửi bởi Le Dinh Hai on 26-07-2015 - 20:47 trong Bất đẳng thức và cực trị

Max nè :

$A=\frac{5x^{2}-7x+1}{\left | 3x-2 \right |+\left | x^{2}+1 \right |}$

    $=\frac{5x^{2}-7x+1}{\left | 3x-2 \right |+ x^{2}+1 }$

    $\leq \frac{5x^{2}-7x+1}{x^{2}+1}$

    $\leq 1 + \frac{4x^{2} - 7x}{x^{2} + 1}$ (1)

Dấu "=" xảy ra $\Leftrightarrow$ $\left |3x - 2 \right |$= 0

                        $\Leftrightarrow$ $3x - 2$ = 0

                        $\Leftrightarrow$ $x = \frac{2}{3}$

Thay $x = \frac{2}{3}$ vào (1) , ta có : A max = -1.

P/s : sr bạn, mình hơi loạn tí :mellow:

Không chặt chẽ rồi bạn ơi :mellow:

Với$x\geq \frac{2}{3}$,ta có:

$A= \frac{5x^{2}-7x+1}{x^{2}-3x-1}=5+\frac{6-22x}{x^2-2.\frac{2}{3}x+\frac{4}{9}+\frac{13}{3}x-\frac{13}{9}}= 5+\frac{6-22x}{\left ( x-\frac{2}{3} \right )^{2}+\frac{13}{3}x-\frac{13}{9}}\leq 5+\frac{6-22.\frac{2}{3}}{\frac{13}{3}.\frac{2}{3}-\frac{13}{9}}=-1$

Dấu "="xảy ra tại $x=\frac{2}{3}$

Tương tự, với $x<\frac{2}{3}$, ta có $A<-1$

Vậy $maxA=-1$ tại $x=\frac{2}{3}$

Còn $min$ thì không có đâu!!!




#575691 Chứng Minh rằng bằng hữu hạn các bước như trên ta có thể thay đổi toàn bộ các...

Đã gửi bởi Le Dinh Hai on 26-07-2015 - 21:13 trong Giải toán bằng máy tính bỏ túi

Bàn cờ HCN m.n ( m,n > 4) được Tô màu trắng đen xen kẽ. Ta thực hiện các bước sau : lấy 1 Thanh 1.k ( k lẻ) sau đó sẽ đổi màu các ô trong Thanh đó bằng màu mà có số ô nhỏ hơn màu kia . Chứng Minh rằng bằng hữu hạn các bước như trên ta có thể thay đổi toàn bộ các ô bàn cờ thành 1 màu

 

Dinh Xuan Hung:Chú ý cách đặt tiêu đề!

Thay đổi từng ô vuông một

Ta chọn màu đen là màu cuối cùng

B1:Chọn các thanh sao cho có màu đen nhiều hơn trắng(k lẻ,đen trắng xen kẽ nên luôn luôn chọn được),thay màu

B2:Làm liên tục B1 theo cả chiều dọc và chiều ngang

Cứ thế màu đen tăng dần và sẽ thay thế màu trắng(đpcm)




#575713 CMR $(1+\frac{1}{a})(1+\frac{1}...

Đã gửi bởi Le Dinh Hai on 26-07-2015 - 21:42 trong Bất đẳng thức và cực trị

Cách khác:

$VT=\prod (1+\frac{1}{3a}+\frac{1}{3a}+\frac{1}{3a})\geq 4^{3}.\sqrt[4]{\frac{1}{(27abc)^{3}}}\geq 64$




#575927 $minA=\sum \frac{a^{2}+2b+1}{c^2+1...

Đã gửi bởi Le Dinh Hai on 27-07-2015 - 19:28 trong Bất đẳng thức và cực trị

Tìm $minA=\sum \frac{a^{2}+2b+1}{c^2+1}$ với$a+b+c=3$ và $a,b,c>0$




#576246 $\left\{\begin{matrix} x^{2}+3y^...

Đã gửi bởi Le Dinh Hai on 28-07-2015 - 17:36 trong Phương trình, hệ phương trình và bất phương trình

Cho $x,y,z\geq 1$.Giải hpt:

$\left\{\begin{matrix} x^{2}+3y^{2}+5z^{2}=18 & \\ 2(x+y+z)^{2}+\prod(x+y)=\sum6xy+8xyz & \end{matrix}\right.$




#576316 CMR: với mọi số nguyên n $\geq 2$ thì $\sqrt[n]...

Đã gửi bởi Le Dinh Hai on 28-07-2015 - 21:43 trong Đại số

BÀI 3: CMR: với mọi số nguyên n $\geq 2$ thì $\sqrt[n]{2}$ là số vô tỉ

giả sử $\sqrt[n]{2}$ là số hữu tỉ 

ta có $\sqrt[n]{2}=\frac{m}{p}$$(m,n\epsilon \mathbb{N},(m,p)=1)$

=>$2p^{n}=m^{n}$

=>$m^{n}\vdots p^{n}$

mà $(m,p)=1$

=>vô lí nên $\sqrt[n]{2}$ là số vô tỉ(đpcm)




#576328 $\left\{\begin{matrix} x^{2}+3y^...

Đã gửi bởi Le Dinh Hai on 28-07-2015 - 22:06 trong Phương trình, hệ phương trình và bất phương trình

Đáp án

Ta có $2(x+y+z)^{2}+\prod (x+y)=\sum 6xy +8xyz$

<=>$2(x+y+z)^{2}+\sum x^{2}y+ \sum xy^{2}+2xyz=\sum 6xy +8xyz$

<=>$2(x+y+z)^{2}+\sum9x +9+\sum3xy+(xy+yz+zx)(x+y+z)=\sum9x +\sum9xy +9+9xyz$

<=>$[2(x+y+z)+3](x+y+z)+(xy+yz+zx)(x+y+z+3)=9\prod (x+1)$

<=>$[\sum(1+x)(1+y)](x+y+z+3)=9\prod (x+1)$

<=>$\frac{\sum(1+x)(1+y)}{\prod (1+x)}=\frac{9}{x+y+z+3}$

<=>$\sum \frac{1}{1+x}=\frac{9}{x+y+z+3}\leq \frac{3}{1+\sqrt[3]{xyz}}$

Lại có $\frac{1}{1+a}+\frac{1}{1+b}\geq \frac{2}{1+\sqrt{ab}}$  (*)

Tương tự chứng ming$\sum \frac{1}{1+x}\geq \frac{3}{1+\sqrt[3]{xyz}}$  (2*)

Từ (*) và (2*) =>$\sum \frac{1}{1+x}=\frac{3}{1+\sqrt[3]{xyz}}$

Dấu $"="$ xảy ra tại $x=y=z$

mà $x^{2}+3y^{2}+5z^{2}=18$

=>$x=y=z=1,5$




#576342 $\left\{\begin{matrix} x^{2}+3y^...

Đã gửi bởi Le Dinh Hai on 28-07-2015 - 22:22 trong Phương trình, hệ phương trình và bất phương trình

Lời giải sai, để chứng minh $\sum \frac{1}{1+x}\geq \frac{3}{1+\sqrt[3]{xyz}}$ cần có thêm điều kiện $xyz \geq 1$ 

$\frac{1}{x+1}+\frac{1}{y+1}\geq \frac{2}{1+\sqrt{xy}}$

<=>$(2+x+y)(1+\sqrt{xy})\geq 2(1+x)(1+y)$

<=>$x\sqrt{xy}+y\sqrt{xy}-2xy\geq 0$

<=>$\sqrt{xy}(\sqrt{x}-\sqrt{y})^{2}\geq 0$(lđ)